NURS 309 Quiz 6 Male Reproductive, CA Exam

Pataasin ang iyong marka sa homework at exams ngayon gamit ang Quizwiz!

What technique is used in oral car for a patient with stomatitis?

"Swish and spit" room-temperature tap water at least four times a day

A patient had an anterior colporrhaphy and is returning to the clinic for the follow up appointment. Which patient statement indicates that the procedure has achieved desired therapeutic outcome?

-"I have good control over my urination"

A 40 y/o woman has heavy vaginal bleeding. Which question is the priority in evaluating the patient's condition?

-"are you feeling weak, dizzy or light headed?"

A patient reports the sensation of feeling as if "something is falling out" along with painful intercourse, backache, and a feeling of heaviness or pressure in the pelvis. Which question does the nurse ask to assess for cystocele?

-"are you having urinary frequency or urgency?"

A young woman reports that she has a genital discharge causing irritation and odor. She feels embarrassed, but insists that she has not had recent sexual relations. Which question is the nurse most likely to ask?

-"have you recently taken any antibiotic medications?"

An obese 59 y/o patient describes excessive menstrual bleeding that occurs approximately every 10 days. Which question should the nurse ask first?

-"how many pads or tampons do you use each day?"

The patient needs diagnostic testing to determine the presence of endometrial thickening and possible cancer. Which brochure will the nurse prepare for this patient?

-"how transvaginal ultrasound and endometrial biopsy are used in cancer diagnosis"

A patient is recently diagnosed with a fibrocystic breast condition and is very fearful that she will develop breast cancer. What does the nurse tell the patient?

-"there is no increased risk for developing breast cancer"

The mother of a 17 y/o adolescent girl tells the nurse that her daughter has been purging, showing anorexic behavior, and continuously exercising. Based on the mother's report, which question related to the reproductive system would the nurse ask the girl?

-"when was your last menstrual period?"

According to the American Cancer Society recommendations, which healthy woman with no previous history of an abnormal pap test should be advised to have a pap test every 3 years?

-21 y/o

The nurse is teaching a class about laparoscopic or hysterscopic myomectomy. Which patient should be invited to attend the teaching session?

-23 y/o woman with uterine fibroids who would like to have children in the future

Which patient has the greatest need for evaluation of possible endometrial cancer?

-63 y/o woman who is having painless vaginal bleeding

Which woman has the highest risk for developing breast cancer?

-68 y/o who takes hormone replacement therapy

What are the side effects of radiation?

-Altered taste sensation -Skin changes and permanent local hair loss -Fatigue

Which factors are used to determine a cancer patient's absorbed radiation dose?

-Intensity of radiation exposure -Proximity of radiation source to the cells -Duration of exposure

Biologic response modifies have which positive effects on patients receiving chemotherapy?

-Less risk for life-threatening infections -Able to tolerate higher doses of chemotherapy

An African American male patient has a prostate- specific antigen (PSA) level less than 2.5 ng/mL. Which information should the nurse give the patient?

-PSA level of less than 2.5ng/mL is generally considered normal

The patient reports fatigue and low libido. Based on the patient's report of symptoms, which laboratory result would the nurse seek out first?

-RBC count

Which laboratory result indicates that the primary goal of treatment of the patient's uterine fibroids has been successful?

-RBC is in normal limits

Which intervention does the nurse implement fora patient receiving chemotherapy to prevent the serious side effects of sepsis and disseminated intravascular coagulation?

-Strict handwashing -Monitor WBCs and clotting factors -Educate patient and family -Frequently assess for signs of infection

Which factors determine the type of therapy for cancer?

-Type and location of cancer -Overall health of the patient -Whether the cancer has metastasized -Family history and genetics

What does a course of chemotherapy normally include?

-Varience with patient's responses to therapy -Timed dosing of the therapy to minimize normal cell damage

The nurse reads in the patient's chart that the HCP is concerned about myelopsuppression. Which lab results will the nurse closely monitor and report to the provider?

-WBC count -RBC count -Platelet count

Which factor makes the mammogram a more sensitive screening tool than other tests?

-able to reveal masses too small to be palpated manually

Which factor is the incidence of breast cancer most likely related to?

-aging

A patient has just been diagnosed with breast cancer and informed that surgery is likely to be needed. The patient seems anxious and upset. What is the priority nursing care for this patient?

-allow patient to openly talk about feelings

Disorders that alter a woman's metabolism or nutrition can result in which condition?

-amenorrhea

A 36 y/o patient is diagnosed with dysfunctional uterine bleeding. During the pelvic exam, the HCP determines that the bleeding is acute. What is the nurse's priority action?

-anticipate an order for oral contraceptive therapy

A patient stopped having menses about a year ago. When does the nurse advise the patient to perform breast self examination?

-any day of the month, but follow a consistent schedule

A patient had a mastectomy several years ago and calls the nurse to report sensations of heaviness, aching, fatigue, numbness, tingling, and swelling in the affected arm and upper chest. Which intervention would the nurse use?

-arrange an immediate appointment with a lymphedema specialist

A patient with uterine leiomyomas reports a feeling of pelvic pressure, constipation, and urinary retention. She says "i cant button my pants anymore". What does the nurse do to further evaluate the patient's condition?

-assess abdomen for distention and enlargement

Three years after the patient was diagnosed and treated for endometrial cancer, the patient and family are told that the cancer is recurring. Which intervention is the nurse most likely to use?

-assess for readiness to explore palliative care and hospice

A patient is receiving external radiation therapy for treatment of endometrial cancer. What task does the nurse delegate to UAP?

-assist pt to ambulate if she feels fatigue or tiredness

The nurse is caring for a patient who had a posterior colporrhaphy. Which task is most appropriate to delegate to UAP?

-assist pt with a sitz bath to relieve comfort

The nurse is giving discharge teaching to a patient who had a transvaginal repair for pelvic organ prolapse using a surgical mesh. What does the nurse include?

-avoid sexual intercourse for 6 wks

A patient had a partial mastectomy. When teaching about care of the arm on the affected side, what does the nurse tell the patient?

-begin using your arm for normal activities, such as eating or combing your hair

A patient has just been diagnosed with advanced breast cancer. Which behavior by the patient is the strongest indicator of readiness for additional patient teaching and information?

-being quiet and thoughtfully fingering the lace on her new bra (lol wtf)

The nurse is preparing patient teaching for several young women who will undergo surgical procedures for gynecologic problems. Which surgical procedure is most likely to induce menopausal symptoms?

-bilateral salpingo-oophorectomy

The nurse is reviewing the laboratory results from a postmenopausal woman being evaluated for a breast mass. What type of metastasis does the increased serum calcium and alkaline phosphatase levels suggest?

-bone

Which diagnostic test is considered the most definitive for diagnosing breast cancer?

-breast biopsy

The nurse encourages a teenage pt to receive the human papillomavirus (HPV) vaccine because it protects against which type of cancer?

-cervical cancer

The nurse is caring for several patients who had total abdominal hysterectomies. All patients are coming to the clinic for their 6 week follow up appointment. Which patient demeanor is the strongest indicator that there is a need for a psychological referral?

-disheveled and lackluster and displays a lack of interest in questions

What preprocedural instructions would the nurse give to the patient about a mammogram?

-do not use lotions, creams or powders on breasts before

A 54 y/o woman has identified a hard breast mass with irregular borders, redness, and edema. She reports a nipple discharge and enlarged axillary nodes. Based on the patient's age and description of the symptoms, what does the nurse suspect?

-ductal ectasia

The nurse is taking a history on a patient with probable gynecologic cancer. Which clinical manifestation is a sign of metastasis?

-dysuria

A patient had a partial mastectomy yesterday and the nurse notes that the patient is very nervous because of removal of breast tissue. What is the nurse's priority intervention?

-encourage pt to discuss her fears and ask questions about her concerns

A patient is diagnosed with uterine leiomyomas. What does the nurse expect to see in the documentation as the patient's chief presenting symptoms?

-heavy vaginal bleeding

A patient with a fever, myalgia, sore throat, and sunburn like rash is admitted with diagnosis of toxic shock syndrome. What additional clinical manifestation should the nurse assess for?

-hypotension

A patient with a fibrocystic breast condition has just undergone fine needle aspiration to drain the cyst fluid and reduce pressure and pain. When would the nurse prepare patient education material about breast biopsy?

-if fluid is not aspirated -if the mammogram shows suspicious findings -if the mass remains palpable after aspiration -if aspirated fluid reveals cancer cells

What is the important information regarding breast cancer surveillance for a patient who had breast augmentation surgery?

-implant displacement allow more complete examination

According to the American Cancer Society guidelines, annual screening mammograms are not recommended for women less than 40 y/o. What is the underlying rationale for this recommendation?

-in younger women, there is little difference in the density of normal tissue and malignant tumors

A patient has just been informed that she has an abnormal pap smear and a positive human papillomavirus test. The nurse should be prepared to provide information about which topic?

-increased risk for cervical cancer

A 23 y/o female has a decreased level of follicle stimulating hormone. What may this finding indicate?

-infertility

A young woman is suspected of having invasive breast cancer. Based on the types and frequencies of breast cancer, what is the most likely diagnosis?

-infiltrating ductal carcinoma

What postprocedure instructions would the nurse give to a patient who had a prostate biopsy?

-light rectal bleeding and blood in the urine or stools is expected for a few days

Which patient is most likely to report problems with impotence?

-patient has diabetes

For a patient with a low testosterone level, what is the patient most likely to report?

-problems w/ sexual performance

A patient received treatment for prostate cancer. Which test is most likely to be ordered to monitor the disease after treatment?

-prostate specific antigen test

The nurse is caring for a patient who is diagnosed with ductal ectasia. What is the primary goal of the nursing care?

-reduce anxiety associated w/ threat of breast cancer

The nurse is giving discharge teaching to a woman who had local cervical ablation. What information would be included?

-report heavy vaginal bleeding or foul smelling discharge

A patient with a fibrocystic breast condition is prescribed hormone therapy to manage symptoms. Which teaching materials will the nurse prepare for the patient related to the therapy?

-s/sx of stroke, liver disease, and increased intracranial pressure

The nurse is talking to a woman who was diagnosed with breast cancer at the age of 30. The woman has a 6 y/o daughter. What advice does the nurse give about breast cancer screenings for the daughter?

-screenings should begin when she turns 20 y/o

A patient had loop electrosurgical excision procedure (LEEP) for treatment and diagnosis of cervical cancer. In the discharge instructions, what does the nurse tell the patient to expect after the procedure?

-spotting

A patient is admitted with toxic shock syndrome. What organism is frequently associated with this syndrome when it occurs as a menstrual related infection?

-staphylococcus aureus

What findings would be considered normal findings for the scrotum of a young white male?

-suspended below the pubic bone -contracts with exposure to cold -sparse hair follicles -skin of pouch is thin walled

What is the primary factor for the low survival rates for patient who are diagnosed with ovarian cancer?

-symptoms are mild and vague, therefore the cancer is often not detected until late stage

For a patient with mild discomfort from a fibrocystic breast condition, what will the nurse teach the patient about self care measures?

-take analgesics and limit salt before menses to decrease swelling

The nurse is teaching self care management to a 39 y/o woman who had an abdominal hysterectomy. Which point would be emphasized to avoid complications of this surgery?

-take temperature twice a day for 3 days after surgery

A male patient had mumps as a child which caused orchitis. Which potential complication could result?

-testicular atrophy

A male patient reports that he has a decreased libido. The nurse assesses for which factors related to this condition?

-tobacco use -alcohol consumption -illicit drug use

The HCP has just informed the patient about the diagnosis and complications of salpingitis. Which intervention is the nurse most likely to use with this patient?

-use empathetic listening for feelings related to possible infertility

The surgical procedure for stage 1 disease of endometrial cancer involves removal of which components?

-uterus -fallopian tubes -ovaries -peritoneum fluid for cytologic examination

The nurse reads in the patient's chart that the patient is experiencing surgical menopause after having a total hysterectomy and bilateral salpingo-oopherectomy. What expected s/sx does the nurse anticipate that the patient will report?

-vaginal changes

A 60 y/o female patient informs the nurse that she has experienced some vaginal change since menopause. What gynecologic change is the patient most likely to report?

-vaginal dryness

What self management strategy would the nurse recommend to a a patient to prevent vulvovaginitis?

-wear breathable fabrics, such as cotton

According to the American Cancer Society, what are the recommendations for early detection by screening for breast masses?

-women aged 45-54 should have an annual mammogram, then every 2 years @ age 55 and older

A patient had undergone external beam radiation therapy (EBRT) for palliative treatment of prostate cancer. What suggestions does the nurse make to help the patient manage acute radiation cystitis secondary to ERBT? A. Limit intake of water and other fluids B. Avoid consumption of caffeinated drinks C. Increase consumption of dairy products D. Wash genitals with mild soap and water

B. Avoid consumption of caffeinated drinks

What are common sites of metastasis for prostate cancer? SATA A. Pancreas B. Bones of the pelvis C. Liver D. Lumbar spine E. Lungs F. Kidneys

B. Bones of the pelvis C. Liver D. Lumbar spine E. Lungs

The nurse is caring for a client who has just returned to the surgical unit after a transurethral resection of the prostate (TURP). Which assessment finding will require the most immediate action? A. Blood pressure reading of 153/88 mm Hg B. Catheter that is draining deep red blood C. Client not wearing anti embolism hose D. Client report of abdominal cramping

B. Catheter that is draining deep red blood

Which is the most important for the nurse to do when providing care to a client who had a transurethral resection of the prostate? A. Maintain patency of the cystostomy tube B. Ensure patency of the indwelling catheter C. Keep the abdominal dressing clean and dry D. Observe the wound for hemorrhage and infection

B. Ensure patency of the indwelling catheter

What is the major advantage to tadalafil compared to other medications or treatments for erectile dysfunction? A. User is able to control erections B. Erection occurs more naturally C. There is no need to abstain from alcohol D. Sexual stimulation is not required

B. Erection occurs more naturally

The nurse is teaching a patient at risk for prostate cancer about food sources of omega-3 fatty acids. Which food does the nurse suggest? A. Red meat B. Fish C. Watermelon D. Oatmeal

B. Fish

A client who had a suprapubic prostatectomy returns from the postanesthesia care unit and accidently pulls out the urethral catheter. What should the nurse do first? A. Reinsert a new catheter B. Notify the health care provider C. Check for bleeding by irrigating the suprapubic tube D. Take no immediate action if the suprapubic tube is draining

B. Notify the health care provider

The nurse is reviewing the laboratory results from a patient being evaluated for lower urinary tract symptoms. What does an elevated prostate-specific antigen (PSA) level and serum acid phosphatase level in this patient indicate? A. Infection B. Prostate cancer C. Benign prostatic hyperplasia D. Infertility

B. Prostate cancer

The nurse is giving instructions to unlicensed assistive personnel (UAP) about hygienic care for an older adult patient who is uncircumcised. What does the nurse instruct the UAP to do? A. Defer cleaning the penis because of patient embarrassment B. Replace the foreskin over the penis after bathing C. Observe the penis and the foreskin for redness or odor D. Avoid touching the foreskin because of hypersensitivity

B. Replace the foreskin over the penis after bathing

Which sign/symptom is associated with advanced prostate cancer? A. Difficulty starting urination B. Swollen lymph nodes C. Frequent bladder infections D. Erectile dysfunction

B. Swollen lymph nodes

A patient tells the nurse that he was diagnosed with benign prostatic hyperplasia. Based on this diagnosis, which symptom is the patient most likely to report? A. Pain in the scrotum B. Trouble passing urine C. Erectile dysfunction D. Constipation

B. Trouble passing urine

The nurse reads in the patient's chart that the patient had a transurethral needle ablation. Which question would the nurse ask the patient to determine if the procedure achieved the intended therapeutic goal? A. "Did the pain resolve completely after the procedure?" B. "Are you able to achieve and sustain an erection?" C. "Have your problems with urination been resolved?" D. "Have you had a follow-up prostate-specific antigen level?"

C. "Have your problems with urination been resolved?"

The nurse is talking to a patient who had the relatively new procedure, prostate artery embolization. Which patient report indicates that the intended goal of therapy has been met? A. "My problem with ejaculation is much better" B. "I used the sperm bank and now I'm less anxious" C. "I am not having any more urinary symptoms" D. "My doctor said the prostate-specific antigen was good"

C. "I am not having any more urinary symptoms"

The nurse is talking with an older patient who has benign prostatic hyperplasia. Which report by the patient requires emergent care? A. "I leak and dribble urine." B. "I have to get up at night to pee." C. "I can't pass my urine today." D. "I am passing dark yellow urine."

C. "I can't pass my urine today."

The nurse teaches a patient with benign prostatic hyperplasia to follow which instructions? SATA A. Take diuretics B. Avoid sexual intercourse C. Avoid antihistamine D. Avoid caffeine E. Avoid drinking large amounts of fluid in a short time F. Void when the urge occurs

C. Avoid antihistamine D. Avoid caffeine E. Avoid drinking large amounts of fluid in a short time F. Void when the urge occurs

A patient who has testicular cancer is likely to have which common problem? A. Priapism B. Erectile dysfunction C. Azoospermia D. Cryptoorchidism

C. Azoospermia

The nurse is reviewing the laboratory results for a patient with prostate cancer. Which laboratory result suggests metastasis to the bone? A. Decreased alpha fetoprotein B. Increased blood urea nitrogen C. Elevated serum alkaline phosphatase D. Decreased serum creatinine

C. Elevated serum alkaline phosphatase

The nurse is teaching a patient who had an open radical prostatectomy about how to manage the common potential long-term complications. What does the nurse teach the patient? A. How to perform testicular self-examination B. How to manage a permanent suprapubic catheter C. How to perform Kegel perineal exercises D. How to use dietary modifications to acidify the urine

C. How to perform Kegel perineal exercises

An older patient reports that he has an enlarged prostate with chronic urinary retention but declines to seek treatment because "it's been that way for a long time." The nurse would encourage a follow-up appointment to prevent which complication of this chronic condition? A. Prostate cancer B. Erectile dysfunction C. Hydronephrosis D. Testicular cancer

C. Hydronephrosis

A client with cancer of the prostate requests the urinal at frequent intervals but either does not void or voids in very small amounts. What does the nurse conclude is most likely the causative factor? A. Edema B. Dysuria C. Retention D. Suppression

C. Retention

What should the nurse do to obtain an accurate urine output for a client with a continuous bladder irrigation? (CBI) A. Measure the contents of the bedside drainage bag B. Stop the irrigation and determine the urine output C. Subtract the volume of irrigant from the total drainage D. Ensure the urine and irrigant drain into two separate bags

C. Subtract the volume of irrigant from the total drainage

The nurse is reviewing prostate-specific antigen (PSA) results for a patient who had a prostatectomy for prostate cancer several months ago. The PSA level is 40 ng/mL. How does the nurse interpret this data? A. At this stage, PSA level of 40 ng/mL is expected B. The cancer was completely removed C. The cancer is most likely recurring D. Prostate irrigation and infection are present

C. The cancer is most likely recurring

Which word best describes the purpose of cytoreductive surgery for cancer?

Control

Which man has the highest risk for prostate cancer? A. A 65-year-old Caucasian American man who has two cousins with prostate cancer B. A 45-year-old Asian American man with a history of benign prostatic hyperplasia C. A 55-year-old Hispanic American man who has poor dietary practices D. A 75-year-old African American man whose brother had prostate cancer

D. A 75-year-old African American man whose brother had prostate cancer

Which nursing action can best prevent infection from a urinary retention catheter? A. Cleansing the perineum B. Encouraging adequate fluid intake C. Irrigating the catheter once daily D. Cleansing around the meatus routinely

D. Cleansing around the meatus routinely

The nurse is using the International Prostate Symptom Score to assess a patient. Which data does the nurse intend to obtain through the use of this assessment tool? A. Patient's attitudes and beliefs about prostate surgery B. Pattern of growth of prostate and correlation with symptoms C. Data in aggregate that can be used for prostate research D. Effect of urinary symptoms on the quality of life

D. Effect of urinary symptoms on the quality of life

A patient had a transurethral resection of the prostate and has a three-way urinary catheter taped to the left thigh. What does the nurse instruct about the position of the left leg? A. Maintain slight abduction B. Maintain slight flexion of the hip C. Keep the leg elevated D. Keep the leg straight

D. Keep the leg straight

The day after a radical prostatectomy, a client has blood clots in the urinary catheter and reports bladder spasms. The client says that his right calf is sore and that he feels short of breath. Which action will the nurse take first? A. Irrigate the catheter with 50 mL of sterile saline B. Administer oxybutynin 5 mg orally C. Apply warm packs to the right calf D. Measure oxygen saturation using pulse oximetry

D. Measure oxygen saturation using pulse oximetry

An older adult patient had a transurethral resection of the prostate at 0800. At 1500, the nurse assess the patient. Which findings does the nurse report to the health care provider? A. Patient reports a continuous urge to void B. Patient keeps attempting to void around catheter C. Patient wants to get out of bed D. Patient keeps moving and ketchup-like urine output is noted

D. Patient keeps moving and ketchup-like urine output is noted

The nurse obtains the following assessment data about a client who has had a transurethral resection of the prostate (TURP) and has continuous bladder irrigation. Which finding indicates the most immediate need for nursing intervention? A. The client states he feels a continuous urge to void B. The catheter drainage is light pink with occasional clots C. The catheter is tape to the client's thigh D. The client reports painful bladder spasms

D. The client reports painful bladder spasms

A patient has a diagnosis of cancer with a gram-negative infections. The nurse assesses bleeding from many sites throughout the body. For which condition does the nurse expect to perform nursing interventions?

Disseminated intravascular coagulation

Which patient with cancer has the greatest risk for infection?

Has leukemia with neutropenia

The patient has breast cancer with bone metastasis. Based on this information, which laboratory result would the nurse expect to see?

Increase in serum calcium level

What does the nurse wear a dosimeter when providing care to a patient receiving brachytherapy?

Measure's the nurse's exposure to radiation

A patient is on a newer protocol, dose-dense chemotherapy. Which factor is most likely to contribute to patient noncompliance if the nurse fails to educate the patient and the family?

Side effects are likely to be more intense and unpleasant

What is the most typical schedule for radiation therapy?

Small doses of radiation given on a daily basis for a set time period

A patient with advanced breast cancer reports severe back pain and leg weakness. Based on these symptoms, what does the nurse suspect?

Spinal cord compression

A patient with lymphoma reports severe facial swelling, tightness of the gown collar, and epistaxis. Which complication does the nurse suspect?

Superior vena cava syndrome

Which therapy is an example of the cornerstone for cancer treatment?

Surgical removal of the visible and microscopic tumor

At what point is a patient radioactive when receiving radiation treatment by tele therapy and is therefore a potential danger to other people?

The patient is never radioactive

The nurse is supervising a nursing student who is giving care to a patient with a sealed implant. The nurse would intervene if the student performed which action?

Wears a lead apron while providing care, but turns away from the patient

What is an action of carcinogens? a. Damage the DNA b. Increase the migration of cells c. Turn off oncogenes d. Stimulate viral activity

a. Damage the DNA

Ideally the health care team should encourage primary prevention measures to target which step of carcinogenesis? a. Initiation b. Promotion c. Progression d. Metastatis

a. Initiation

The patient has breast cancer with bone metastasis. Based on this information, which laboratory result would the nurse carefully monitor? a. Serum calcium level b. Serum blood glucose c. Serum potassium level d. Serum sodium level

a. Serum calcium level

The nurse is preparing a brochure to inform patients about secondary prevention of cancer. Which information would be included? a. Yearly mammography for women starting at the age of 45 b. Chemoprevention with vitamin therapy c. Removing colon polyps for cancer prophylaxis d. Using sunscreen and had when outdoors

a. Yearly mammography for women starting at the age of 45

What information can be obtained by surgical staging? SATA a. assessment of tumor size b. number of tumors c. sites of tumors d. types of tumors e. pattern of spread of tumors f. pain related to tumors

a. assessment of tumor size b. number of tumors c. sites of tumors e. pattern of spread of tumors

The nurse is aware that the most common way for cancer to spread is bloodborne metastasis. In caring for a patient with cancer, what type of precautions would the nurse use? a. contact isolation precautions b. standard precautions c. neutropenic precautions d. droplet precautions

b. standard precautions

What role do normal hormones and proteins such as insulin and estrogen play in the development of cancer? a. they prolong or delay the growth of cancer cells b. they can promote frequent division of cells c. they act like carcinogens under certain conditions d. they turn off the suppressor genes

b. they can promote frequent division of cells

Which woman would be the most likely candidate to consider removal of "at risk" breast tissue? a. Has a family history of breast and colon cancer and eats a high-fat diet b. Has large breasts that make self-examination difficult and smokes cigarettes c. Has mutations in the BRCA1 and BRCA2 genes and sister had breast cancer d. Has mammogram results that suggest an immediate biopsy is needed

c. Has mutations in the BRCA1 and BRCA2 genes and sister had breast cancer

A patient is taking oprelvekin. Which assessment data finding indicates that the therapy is working? a. Weight has increased by 2 lbs b. Nausea and vomiting are relieved c. Platelet count is increasing d. Hemoglobin level is normalizing

c. Platelet count is increasing

If a primary tumor is located in a vial organ, what happens? a. Cancer is more likely to spread to other sites b. The organ stops producing normal cells c. There is interference with organ function d. Function of the organ is initially increased

c. There is interference with organ function

The nurse sees that the patient is taking tamsulosin. Which question would the nurse ask to determine if the medication is achieving the desired therapeutic effect? A. "Are you still having trouble passing urine?" B. "Does your urine have a strong odor or appear cloudy?" C. "Are you having any problems with achieving an erection?" D. "Have you had a green or yellow discharge from your penis?"

A. "Are you still having trouble passing urine?"

An older patient's wife is very upset because "my husband was just told he had prostate cancer. He feels fine now, but the doctor told him to watch and wait. Why are we just watching? What are we supposed to do?" What is the nurse's best response? A. "Prostate cancer is slow growing. Your husband needs regular prostate specific antigen testing; I'll give you a list of symptoms to watch for." B. "This is very upsetting news. Let's sit down and talk about how you feel and then I will have the doctor talk to you again." C. "It's okay, don't be upset. This is a very common way to handle prostate cancer for men who are your husband's age." D. "I can get you some information about prostate cancer. this will help you understand why the doctor said this to your husband."

A. "Prostate cancer is slow growing. Your husband needs regular prostate specific antigen testing; I'll give you a list of symptoms to watch for."

The emergency department nurse receives change-of-shift report about four clients. Which one should be assessed first? A. A 19-year-old client with scrotal swelling and severe pain that has not decreased with elevation of scrotum B. A 25-year-old client who has a painless indurated lesion on the glans penis C. A 44-year-old client with an elevated temperature, chills, and back pain associated with recurrent prostatitis D. A 77-year-old client with abdominal pain and acute bladder distension

A. A 19-year-old client with scrotal swelling and severe pain that has not decreased with elevation of scrotum

Which conditions meet the criteria for having a surgical intervention for benign prostatic hyperplasia? SATA A. Acute urinary retention B. Hydronephrosis C. Acute urinary tract infection that does not respond to first-line antibiotics D. Recurrent kidney stones E. Hematuria F. Chronic urinary tract infections secondary to residual urine in the bladder

A. Acute urinary retention B. Hydronephrosis E. Hematuria F. Chronic urinary tract infections secondary to residual urine in the bladder

A young patient is diagnosed with testicular cancer. He and his wife have been trying to conceive a child for several months. What information does the nurse give the couple about sperm storage? A. Arrangements for sperm storage should be made as soon as possible after diagnosis B. Sperm collection should be completed after radiation therapy or chemotherapy C. Two or three samples should be collected 6 days apart D. Saving sperm helps to alleviate fears related to erectile dysfunction

A. Arrangements for sperm storage should be made as soon as possible after diagnosis

A client had a suprapubic prostatectomy. Which type of tube can the nurse expect the client to have when he returns to his room from the postanesthesia care unit? A. Cystostomy B. Nasogastric C. Nephrostomy D. Ureterostomy

A. Cystostomy

The nurse is teaching a patient who had an open retroperitoneal lymph node dissection. What instructions does the nurse give to the patient? SATA A. Do not lift anything over 15 lb B. Limit intake of fluids to 1000 mL per day C. Do not drive a car for several weeks D. Perform monthly testicular self-examination on the remaining tests E. Have follow-up diagnostic testing for at least 3 years after the surgery F. Report fever, drainage, or increasing tenderness or pain around the incision

A. Do not lift anything over 15 lb C. Do not drive a car for several weeks D. Perform monthly testicular self-examination on the remaining tests E. Have follow-up diagnostic testing for at least 3 years after the surgery F. Report fever, drainage, or increasing tenderness or pain around the incision

During the first 24 hours after prostatectomy, what is the priority concern? A. Hemorrhage B. Infection C. Hydronephrosis D. Confusion

A. Hemorrhage

A nurse is designing a teaching plan for a patient with an enlarged prostate with obstructive symptoms. What action could the patient preform that might help to relieve the obstruction? A. Increase frequency of sexual intercourse B. Void before going to bed and upon waking C. Urinate forcefully after drinking fluids D. Spread fluid intake throughout the day

A. Increase frequency of sexual intercourse

What are common serum tumor markers that confirm a diagnosis of testicular cancer? SETA A. Lactate dehydrogenase B. Early prostate cancer antigen C. Glutathione S-transferase D. Alpha-fetoprotein E. Beta human chorionic gonadotropin F. BRCA1 or BRCA2 mutations

A. Lactate dehydrogenase D. Alpha-fetoprotein E. Beta human chorionic gonadotropin

A patient had a transrectal ultrasound with biopsy earlier in the day. What urine characteristics does the nurse expect to see? A. Light pink urine B. Bright red urine C. Dark urine with small clots D. Very pale yellow urine

A. Light pink urine

The nurse is taking a health history on a patient with organic erectile dysfunction. What are possible causes of this condition? SATA A. Medications for hypertension B. Obesity C. Thyroid disorders D. Diabetes mellitus E. Diverticulitis F. Smoking and alcohol

A. Medications for hypertension C. Thyroid disorders D. Diabetes mellitus F. Smoking and alcohol

The patient is prescribed a broad-spectrum antibiotic for prostatitis. Which laboratory result indicates that the medication is having the desired therapeutic effect? A. Normalization of white cell count B. Decreased blood urea nitrogen level C. Increased red blood cell count D. Prostate-specific antigen within normal limits

A. Normalization of white cell count

An older patient is scheduled for an annual physical including a prostate-specific antigen (PSA) and a digital rectal examination (DRE). How are these two tests scheduled for the patient? A. PSA is drawn before the DRE is performed B. DRE is done several weeks before the PSA C. PSA is reviewed first because DRE may be unnecessary D. Both tests can be done at the convenience of the patient

A. PSA is drawn before the DRE is performed

The nurse is caring for a patient who had minimally invasive surgery for testicular cancer. The nurse is also caring for a patient who had an open radical retroperitoneal lymph node dissection for testicular cancer. The nurse anticipates that the second patient has greater risk for which conditions? A. Paralytic ileus B. Urinary incontinence C. Lower urinary tract symptoms D. Fluid overload

A. Paralytic ileus

The nurse is giving discharge instructions to a patient who had a transurethral resection of the prostate. What does the nurse include in the instructions? A. Reassurance that loss of control of urination or dribbling of urine is temporary B. Instructions about how to apply a condom catheter and monitor for skin breakdown C. Advice about how to control bleeding and passage of blood clots D. Information about the side effects related to aminocaproic acid

A. Reassurance that loss of control of urination or dribbling of urine is temporary

After a radical prostatectomy, a client is ready to be discharged. Which nursing action included in the discharge plan should be assigned to an experienced LPN/LVN? A. Reinforcing the client's need to check his temperature daily B. Teaching the client how to care for his retention catheter C. Documenting a discharge assessment in the client's chart D. Instructing the client about the prescribed narcotic analgesic

A. Reinforcing the client's need to check his temperature daily

A patient had a transrectal ultrasound with biopsy. After this procedure, what does the nurse instruct the patient to do? A. Report fever, chills, bloody urine, and any difficulty voiding B. Limit fluid intake for several hours after the procedure C. Expect decreased urine output for 24 hours after the procedure D. Expect some mild perineal and abdominal pain

A. Report fever, chills, bloody urine, and any difficulty voiding

The advanced practice nurse is performing a testicular exam on a young Caucasian male patient. The practitioner finds a hard, painless lump. This finding is considered the most common manifestation of which disease or disorder? A. Testicular cancer B. Erectile dysfunction C. Prostate cancer D. Epididymitis

A. Testicular cancer

A patient has an enlarged prostate. Which procedure does the nurse anticipate that the health care provider will order to test for bladder outlet obstruction? A. Urodynamic pressure-flow study B. Bladder scan C. Transrectal ultrasound D. Computer tomography scan

A. Urodynamic pressure-flow study

A nurse is caring for a client with an indwelling urinary catheter. What is the most important action for the nurse to implement when irrigating the bladder? A. Use sterile equipment B. Instill the fluid under high pressure C. Warm the solution to body temperature D. Aspirate immediately to ensure return flow

A. Use sterile equipment

The nurse hears in report that the patient has cachexia. Which assessment will the nurse plan to perform?

Appetite and nutritional intake

A patient with cancer tells the nurse she had numbness and weakness in her legs. What is the nurse's best response?

Are you having any back pain?

Each chemotherapeutic agent has a specific nadir. What is important to do when giving combination therapy?

Avoid giving agents with similar nadirs at the same time

The nurse is talking to a 35-year-old African American man about prostate-specific antigen (PSA) testing. The patient tells the nurse that his father was diagnosed with prostate cancer in his 50s. What should the nurse tell the patient? A. "Although authorities do not always agree, PSA testing usually starts at age 50" B. "Your genetic and racial risk factors suggest testing should begin at age 45." C. "Because of our African American heritage, you should start testing now" D. "PSA testing can be started at any time for all males at any age"

B. "Your genetic and racial risk factors suggest testing should begin at age 45."

The nurse is caring for a patient who is one day post op for a total abdominal hysterectomy. Which assessment finding is cause for great concern?

-pt reports saturating one pad in an hour w/ dark red blood

A patient who had surgery for breast cancer appears in need of continued community support. The nurse refers the patient to which organization?

-reach to recovery

The patient had a transurethral resection of the prostate (TURP) several days ago, and the urinary catheter was removed 6 hours ago. Which sign/symptom must be resolved before the patient is discharged? A. Patient has not voided since the catheter was removed B. Patient reports a burning sensation with urination C. Patient reports dribbling and leakage since catheter was removed D. Patient reports anxiety related to sexual function because of TURP

A. Patient has not voided since the catheter was removed

A nurse is caring for a client with a diagnosis of benign prostatic hyperplasia (BPH). Which information about this condition is important for the nurse to consider when caring for the client? A. It is a congenital abnormality B. A malignancy usually results C. It predisposes to hydronephrosis D. An increase in the acid phosphatase level occurs

C. It predisposes to hydronephrosis

The patient has an indwelling catheter in place following a transurethral resection of the prostate. What instructions will the nurse give to the unlicensed assistive personnel regarding the catheter? A. Secure the catheter so there is no tension B. Irrigate the catheter to prevent clotting C. Maintain traction on the catheter D. Defer catheter care until the patient is discharged

C. Maintain traction on the catheter

A patient diagnosed with bone cancer reports fatigue, loss of appetite, and constipation. Which lab results does the nurse report immediately?

Calcium level of 10.5 mEq/dL

The patient is having nausea and vomiting, so the nurse checks the medication orders for an antiemetic. The orders indicate to give Avandemet (rosiglitazone maleate and metformin hydrochloride) as needed (PRN) for nausea and vomiting. What should the nurse do?

Contact the provider clarification, because Avandemet is not an antiemetic

A client with benign prostatic hyperplasia has a new prescription for tamsulosin. Which statement about tamsulosin is most important to include when teaching this client? A. "This medication will improve your symptoms by shrinking the prostate" B. "The force of your urinary stream will probably increase" C. "Your blood pressure might decrease as a result of taking this medication" D. "You should avoid sitting up or standing up too quickly"

D. "You should avoid sitting up or standing up too quickly"

A client is admitted to the hospital with a tentative diagnosis of urinary retention related to benign prostatic hyperplasia. There is a secondary diagnosis of delirium related to urosepsis. The health care provider prescribes the insertion of an indwelling urinary retention catheter. What nursing action is most important for the client's safety? A. Secure an order for wrist restraints B. Orient the client to time, place, and person C. Involve family members in the client's care D. Determine if any unsafe behavior patterns exist

D. Determine if any unsafe behavior patterns exist

A patient is prescribed leuprolide, a luteinizing hormone-releasing hormone agonist, for treatment of a prostate tumor. What possible side effect of this medication does the nurse advise the patient about? A. Nipple discharge B. Scrotal enlargment C. Fragility of the skin D. Erectile dysfunction

D. Erectile dysfunction

A patient reports having erectile dysfunction and is seeking a prescription for sildenafil. Because of the potential for dangerous drug-drug interactions, the nurse asks the patient specifically if he takes which type of medication? A. NSAIDs B. Nitrates C. Opioids D. Antilipemics

B. Nitrates

The health care provider tells the nurse that the patient needs testing for prostatitis. Which specimen needs to be obtained and sent to the laboratory? A. Blood sample for serum creatinine B. Prostatic fluid for culture and sensitivity C. Semen sample to test for sperm count D. Blood sample for prostate specific antigen

B. Prostatic fluid for culture and sensitivity

What is the major side effect that limits the dose of chemotherapy?

Bone marrow suppression

The nurse is interviewing a 56 y/o woman who reports irregular and decreased flow of menses for several months. Which question does the nurse ask this patient?

-"are you having any discomfort during intercourse?"

A 20 y/o woman is being evaluated for possible toxic shock syndrome. What question would the nurse ask?

-"do you use internal contraceptives?"

Which patient is most likely to require an iron supplement?

-32 y/o female with heavy menstrual bleeding and an intrauterine device

A patient has had a posterior colporrhaphy. What is indicated in the nursing care of this patient?

-give pain medication before a bowel movement

Which combination of screening techniques is best for early detection of breast cancer?

-mammogram, clinical breast exam, and breast self awareness

A patient had a pelvic examination and needs an additional diagnostic test for possible utereine leiomyomas. The nurse prepares the patient for which diagnostic test?

-monitoring for bleeding

A nurse is caring for a client with a diagnosis of cancer of the prostate. Which serum level should the nurse teach the client to have monitored to follow the course of the disease? A. Serum creatinine B. Blood urea nitrogen C. Nonprotein nitrogen D. Prostate-specific antigen

D. Prostate-specific antigen

The nurse is caring for several patients who are receiving chemotherapy. Which patient is the most likely to need transfer to the intensive care unit?

Patient receiving interleukin therapy for renal cell carcinoma develops edema

Based on the "inverse square law" for radiation exposure, which patient received the smallest radiation dose?

Received radiation dose at a distance of 3 meters

The nurse hears in report that the patient had xerostomia. Which teaching point does the nurse plan to review with the patient?

Regular dental visits are essential because of increased risk for dental caries

Which example best illustrates appropriate prophylactic cancer surgery?

Removal of breast tissue for strong family history of breast cancer

A patient is lying in bed after a mastectomy. How does the nurse position the patient? a. Head of the bed up at least 30* with the affected arm elevated on a pillow b. Supine body position with the affected arm positioned straight by the side c. Any position that is the most comfortable to the patient d. Side-lying position with the unaffected side down toward the mattress

a. Head of the bed up at least 30* with the affected arm elevated on a pillow

According to the American Cancer Society, what are the recommendations for early detection by screening for breast masses? a. Women aged 45-54 should have an annual mammogram, then every 2 years at age 55 and older b. High-risk women should have biannual mammograms and MRI c. High-risk women should be screened for breast cancer annually starting age 21 d. Women aged 60 years or older should have a mammogram every 10 years

a. Women aged 45-54 should have an annual mammogram, then every 2 years at age 55 and older

The nurse reads a laboratory report indicating that the tissue sample of a patient is essentially neoplastic. How does the nurse interpret this report? a. cell growth is abnormal and not needed for tissue replacement b. the tissue specimen shows malignant cell growth c. the parent cell was abnormal, but new growth is benign d. early cell death is inevitable because the morphology is abnormal

a. cell growth is abnormal and not needed for tissue replacement

What is an action of carcinogens? a. damage the DNA b. increase migration of cells c. turn off oncogenes d. stimulate viral activity

a. damage the DNA

Ideally, the health care team should encourage primary prevention measures to target which step of carcinogenesis? a. initiation b. promotion c. progression d. metastasis

a. initiation

Which patient has benign tumor cells that are the result of a small problem with cellular regulation? a. patient is diagnosed with uterine fibroids b. patient is advised that she has melanoma c. patient is advised that he has a G1 tumor d. patient is diagnosed with hair cell leukemia

a. patient is diagnosed with uterine fibroids

A patient is receiving internal radiation therapy (brachytherapy) and has had a low-dose radiation seed implanted directly into the prostate gland. What nursing implication is related to this therapy? A. Ensure that any staff member or visitor who is pregnant is not exposed to the patient B. Organize the nursing care so that exposure to the patient is limited to a few minutes C. Instruct all staff that all urine specimens should be immediately discarded D. Teach the patient that fatigue is common but should pass after several months

D. Teach the patient that fatigue is common but should pass after several months

The advanced-practice nurse is preparing to examine a patient's prostate gland. Before the exam, what does the nurse tell the patient? A. He may feel the urge to defecate or faint as the prostate is palpated B. He should lie supine with knees bent in a fully flexed position C. The examination is very painful, but it only lasts a few seconds D. The gland will be massaged to obtain a fluid sample for possible prostatitis

D. The gland will be massaged to obtain a fluid sample for possible prostatitis

The patient has thyroid cancer and will be treated with injection of the radionuclide iodine-131 (brachytherapy). Which guideline is the most relevant to correctly instructing the UAP about assisting the patient with hygiene and activities of daily living (ADLs)?

Institutional policies for handling body fluids and wastes

Chemotherapy drug dosage is based on total body surface area (TBSA); therefore, what assessment will the nurse perform?

Measure the patient's height and weight

The nurse is assessing a patient who is undergoing outpatient therapy for breast cancer. Which patient report causes the greatest concern because of possible metastasis to a common site? a. "I don't seem to have a very good appetite" b. "My ribs hurt but I haven't had any injuries" c. "My skin is dry and it feels itchy and irritated" d. "I feel like I need to urinate all of the time"

b. "My ribs hurt but I haven't had any injuries"

What is the minimum size for a detectable tumor? a. 1 millimeter b. 1 centimeter c. Depends on type of tumor d. Depends on site of tumor

b. 1 centimeter

Benign cells have which characteristics? Select all a. Contain few pairs of chromosomes b. Resemble the parent tissue c. Growth is orderly with normal growth patterns d. Perform their differentiated function e. Invade other tissues f. Continue to make fibronectin

b. Resemble the parent tissue c. Growth is orderly with normal growth patterns d. Perform their differentiated function f. Continue to make fibronectin

Which lunch tray represents a diet that would decrease the risk of cancer? a. Plain chicken breast on white bread b. Vegetable plate with a bran muffin c. Grilled cheese sandwich with fruit salad d. Bacon cheeseburger with french fries

b. Vegetable plate with a bran muffin

What laboratory result is the most important in relation to the nadir for a chemotherapeutic agent? a. RBC count b. WBC count c. Platelet count d. Serum calcium level

b. WBC count

If apoptosis is occurring within a patient's body, what is the expected outcome of this physiologic process? a. rapid growth of malignant tumors metastasizing through the body b. organs have an adequate number of cells at their functional peak c. normal tissue continues to function in an abnormal place d. cells will initially resemble parent cells but will rapidly mutate

b. organs have an adequate number of cells at their functional peak

A patient is being discharged with a prescription for tamoxifen to decrease the chance of breast cancer recurrence. Because of the common side effect, what does the nurse suggest to the patient? a. Have soda crackers and ginger ale on hand b. Install a hand rail around the bathtub c. Purchase a scale to monitor body weight d. Buy a soft-bristle toothbrush

c. Purchase a scale to monitor body weight

Which woman would be the most likely candidate to consider removal of "at-risk" breast tissue? a. has a family hx of breast and colon cancer and eats a high-fat diet b. has large breasts that make self-examination difficult and smokes cigarettes c. has mutations in the BRCA1 and BRCA1 genes and sister had breast cancer d. has mammogram results that suggest an immediate biopsy is needed

c. has mutations in the BRCA1 and BRCA1 genes and sister had breast cancer

If a primary tumor is located in a vital organ, what happens? a. cancer is more likely to spread to other sites b. the organ stops producing normal cells c. there is interference with organ function d. function of the organ is initially increased

c. there is interference with organ function

Which person has the greatest risk for developing cancer? a. 10 year old african american with allergic asthma b. 32 year old asian immigrant with low income c. 23 year odl white american who has type 1 diabetes d. 62 year old african american who had an organ transplant

d. 62 year old african american who had an organ transplant

The nurse is teaching a patient who is being discharged after having a total abdominal hysterectomy. Which conditions does the nurse tell the pt to immediately report to the surgeon?

-vaginal drainage that becomes thicker or foul smelling -temperature over 100 degrees -burning during urination -pain, tenderness, redness, or swelling in calves

If the function of normal vaginal flora is disrupted, the female patient is most likely to experience what kind of condition?

-vaginal infection

The nurse sees that a patient has been advised by the HCP to apply lindane to the affected area. What is a self care measure for this patient to ensure that the symptoms do not return after using the medication?

-wash clothes & linens, and disinfect the home environment

A patient has undergone a total hysterectomy w/ vaginal repair. Which OTC product will the nurse recommend to decrease sexual discomfort related to intercourse?

-water based lubricants

Which biologic process demonstrates that there is a problem with cellular regulation? a. living cells spend most of their time in G0 state b. mitosis occurs to replace damaged tissue c. cyclin activity is balanced by suppressor genes d. cells continue to divide without contact inhibition

d. cells continue to divide without contact inhibition

Which patient report should be investigated as one of the seven warning signs of cancer? a. soreness and stiffness to joints in the morning b. abdominal pain related to irregular meals c. redness to skin with pain after sun exposure d. sore on nipple present for several months

d. sore on nipple present for several months

Which features are specific to cancer cells? a. they grow very slowly but eventually harm the body b. they have a small, fragile nucleus that is easily damaged c. they produce fibronectin, which strengthens the cell wall d. they have an unlimited life span and spread easily

d. they have an unlimited life span and spread easily

The nurse notes bright-red blood with numerous colts in the urinary drainage bag for a patient who had a transurethral resection of the prostate. Besides notifying the surgeon, what is the nurse's best action? A. Irrigate the catheter with normal saline per protocol B. Remove the urinary catheter and save the tip to culture C. Start an IV infusion and draw blood for type and cross D. Empty the drainage bag and record the appearance of output

A. Irrigate the catheter with normal saline per protocol

The patient has a continuous bladder irrigation via a three-way urinary catheter. At 0700, the urine drainage bag was emptied and 1,000 mL of irrigation fluid was hung. At 1100, 350 mL of irrigation fluid has been administered through the catheter. The urinary drainage bag now contains 600 mL. How many mL of urine has the patient produced in the past 4 hours? ________ mL

250 mL

The nurse is interviewing a patient to determine the presence of lower urinary tract symptoms associated with benign prostatic hyperplasia. Which questions would the nurse ask? SATA A. "Do you have difficulty starting and continuing urination?" B. "Have you ever had a testicular infection?" C. "Do you have reduced force and size of the urinary stream?" D. "Have you noticed dribbling or leaking after urinating?" E. "How many times do you have to get up during the night to urinate?" F. "Have you noticed blood at the start or at the end of urinating?"

A. "Do you have difficulty starting and continuing urination?" C. "Do you have reduced force and size of the urinary stream?" D. "Have you noticed dribbling or leaking after urinating?" E. "How many times do you have to get up during the night to urinate?" F. "Have you noticed blood at the start or at the end of urinating?"

A patient needs surgical intervention for an enlarged prostate but also needs to maintain his anticoagulant therapy. Which brochure would be the most appropriate to prepare for the patient? A. "Talking to Your Doctor About Holmium Laser Enucleation of the Prostate" B. "Transurethral Resection of the Prostate: The Traditional Treatment" C. "Is Laparoscopic Radical Prostatectomy with Robotic Assistance Right for You?" D. "Common Questions About the Open Surgical Technique for Radical Prostatectomy"

A. "Talking to Your Doctor About Holmium Laser Enucleation of the Prostate"

A patient is undergoing large-volume bladder irrigation. During and after the procedure, the nurse observes the patient for confusion, muscle weakness, and increased gastrointestinal motility related to which potentially adverse effect of large-volume irrigation? A. Hyponatremia B. Hypovolemia C. Hypokalemia D. Hypotension

A. Hyponatremia

The nurse is teaching a patient about self-care following an open radical prostatectomy. What does the nurse include in the health teaching? SATA A. Teach how to care for the indwelling catheter and manifestations of infection B. Instruct to walk short distances C. Instruct to have prostate-specific antigen testing 12 weeks after surgery and then once a year D. Advise to maintain an upright position and not walk bent or flexed E. Advise to shower rather than soak in a bathtub for the first 2-3 weeks F. Teach to use enemas or laxatives as needed to prevent straining

A. Teach how to care for the indwelling catheter and manifestations of infection B. Instruct to walk short distances D. Advise to maintain an upright position and not walk bent or flexed E. Advise to shower rather than soak in a bathtub for the first 2-3 weeks

A patient is being discharged with a prescription for oral cancer agent. Based on recent research studies, which teaching point will the nurse emphasize?

Adherence to therapy schedule is more of a problem, but do not skip or reduce doses

A client who has just returned to the surgical unit after a transurethral resection of the prostate (TURP) reports acute bladder spasms. In which order will the nurse perform these prescribed actions? A. Administer acetaminophen/oxycodone 325 mg/5 mg B. Irrigate the retention catheter with 30 to 50 mL of sterile normal saline C. Infuse 500 mL of 5% dextrose in lactated Ringer's solution over 2 hours D. Offer the client oral fluids to at least 2500 to 3000 mL/day

B, A, C, D

The nurse is caring for a patient who must receive an IV chemotherapy infusion. What is the most important intervention related to extravasation?

Frequently monitor the access site to prevent leakage of large volumes

Which cancer patient is the most likely candidate for palliative surgery?

Has continuous vomiting because tumor is obstructing the gastrointestinal (GI) tract

Which cancer patient is the most likely candidate for reconstructive surgery?

Has significant scarring of the face and neck after completing treatments

The nurse hears in report that the patient is distressed by the prospect of developing alopecia. Which question is the nurse most likely to ask to assess the patient's concerns?

How would losing your hair affect your life and activities?

The nurse is talking to a young athlete who needs lung removal for treatment of lung cancer. Which statement best indicates the the patient is coping with the uncertainty of cancer and long-term impact on his physical activities?

I love to compete in sports, but I like to do a lot of other things too

An older adult is having frequent and severe chemotherapy-induced nausea and vomiting (CINV) which seems to be anticipatory and acute. Which assessment is the most important to make?

Signs of dehydration or electrolyte imbalance

How does the nurse apply the "inverse square law" in caring for a patient with cancer who is treated with a radiation implant?

Stands at a distance from the patient as much as possible

Which laboratory result is the most important in relation to the nadir?

White blood cell count

A patient who had a total abdominal hysterectomy is anxious to resume her activities because she has young children at home. What postprocedure information does the nurse provide to the patient?

-avoid sitting for prolonged periods -do not lift anything heavier than 5-10 lbs -when sitting, do not cross legs

A patient in the medical surgical unit says to the nurse "my doctor told me i have advanced breast cancer and i want to give you this bracelet because you have been so sweet to me today". What does the nurse do first?

-sit w/ the patient and allow her to take the lead in the conversation

Which s/sx detected during clinical breast examination suggests advanced breast disease?

-skin change of peau d'orange

A patient is one day post-op after a mastectomy and is anxious to begin the prescribed exercises. Which exercise is appropriate for the patient's first efforts?

-squeeze the affected hand around a soft, round object

What does the nurse instruct the patient to do before a scheduled breast augmentation surgery?

-stop smoking -avoid taking nsaids -avoid taking ginseng

A young woman had minimally invasive surgery for the removal of uterine fibroids. The nurse emphasizes that this information should be included when giving health history; however for which future scenario is the history most essential?

-becomes pregnant and is looking forward to home delivery with midwife assistance

The nurse is examining the breasts of an older woman. What would be considered a normal finding?

-breasts are atrophied, flattened, and elongated

Women who have a personal history of breast cancer are at high risk for developing a recurrence. Which factors contribute to recurrence?

-strong family history of breast cancer -BRCA 2 genetic mutation -BRCA 1 genetic mutation

The nurse is taking a history on a patient who has risk for problems related to reproductive health. Despite the nurse's best attempts to establish rapport and trust, the patient absolutely refuses to answer the questions about sexual practices. What should the nurse do?

-respect the patient's choice to refuse to answer questions

Cancer surveillance for high risk women is used to detect cancer in its early stages and is referred to as what kind of prevention?

-secondary

What is the major advantage of neoadjuvant therapy?

-shrinkage of tumor allows lumpectomy rather than mastectomy

Which intervention would be used for a patient after a modified radical mastectomy?

-teach s/sx of infection and how to monitor for altered wound healing

The nurse is reviewing discharge instructions for a patient who had breast augmentation surgery. What does the nurse include in the instructions?

-breasts will feel tight and sensitive, the breast skin may feel warm and itchy -anticipate having difficulty raising the arms over the head -walk every few hours to prevent DVT -expect some swelling for 3-4 weeks after surgery

In recalling dietary intake for a recent 24 hr period, a female pt describes eating eggs, whole milk, and bacon for breakfast; fried chicken and french fries for lunch; three cheese pizza and ice cream for dinner. This type of diet places her at increased risk for which disorder?

-cancer of the ovaries

Young women who have intercourse as teenagers and/or have multiple sex partners are at high risk for which disease/disorder?

-cervical cancer

In a patient with a reproductive health problem, what health and lifestyle habits would the nurse assess?

-diet -exercise pattern -sleep pattern -sexual practices

The nurse reads in the patient's chart that there is a history of blockage in the fallopian tubes. Based on this history, what is the patient most likely to report?

-difficulty conceiving

The nurse is interviewing a young woman who is considering the option of uterine artery embolization for the treatment of uterine fibroids. Which question would the nurse ask to assist the patient in making a decision?

-"what did the health care provider tell you about the procedure?"

For women with genetic risk factors for breast cancer, which intervention would address one of the modifiable risk factors?

-discuss strategies to avoid weight gain and obesity

The patient needs to be scheduled for an endometrial biopsy to assess unusually heavy menstrual bleeding. Which question is most important to ask, in relation to scheduling the examination?

-"what was the date of your last menstrual period and are you regular?"

Which patient should be advised to have an annual clinical breast examination?

-42 y/o woman who is not having any symptoms

Which patient is most likely to be accepting of surgery and demonstrate better coping behaviors?

-44 y/o woman w/ 2 children and supportive partner had an abdominal hysterectomy for uterine fibroids

Which woman is at greatest risk for pelvic organ prolapse?

-48 y/o obese mother of 4 children

The nurse is helping a patient schedule an appointment for a hysterectomy. When should the procedure be done?

-5 days after menses have ceased

A nurse is working in an ambulatory clinic. Which patient is most likely to need a pelvic examination?

-62 y/o who reports resumption of menses

Based on risk factors and personal history, which woman has the greatest risk of developing breast cancer?

-physician, age 56, who had her first child @ age 38

The HCP tells the nurse that the patient is being evaluated for galactorrhea and to please call with the relevant laboratory results. Which laboratory result will the nurse look for?

-prolactin level

A patient asks the nurse why untreated cancers cause gastrointestinal problems. Which responses are appropriate?

-A tumor in the bowel can decrease your ability to absorb necessary nutrients -Changes in taste can decrease appetite or cause food aversions -Tumors can increase metabolic needs which you are unable to meet

For a patient undergoing external radiation therapy, what do the nurse's instructions include?

-Do not remove the markings -Do not use lotions or ointments -Use mild soap and water on the affected skin

The nurse is caring for a patient with a radioactive implant in the uterus. Which instruction will the nurse give to UAP?

-pt is on bedrest and excessive movement is restricted

A patient is being discharged with a prescription for tamoxifen to decrease the chance of breast cancer recurrence. Because of the common side effect, what does the nurse suggest to the patient?

-purchase a scale to monitor body weight

The health care provider informs the nurse that is likely that the patient's cancer has invaded the bone marrow. Based on this information, the nurse would will be vigilant for which signs and symptoms?

-Fatigue and Weakness -Decreasing white blood cell counts -Bruises or other bleeding signs

A patient is lying in bed after a mastectomy. How does the nurse position the patient?

-HOB at least 30 degrees with the affected arm elevated on a pillow

A patient had a total abdominal hysterectomy. What pt behavior is the best indicator that she is coping and adapting successfully?

-asks questions about the wound care but seems reluctant to do self care

A patient is prescribed traztuzumab for breast cancer. What is the priority nursing intervention for this patient?

-assess for sx of cardiotoxicity

Following a uterine embolization using a vascular closure device, what patient care would the nurse provide?

-assist pt to ambulate 2 hours after the procedure -raise HOB -assess pain level and provide analgesics as needed

The nurse is caring for a patient who had a right sided modified radical mastectomy. Which task does the nurse delegate to the UAP?

-assist pt to ambulate day after surgery

After surgery, a female patient has been told her breast tumor contained estrogen receptors. What is the clinical significance of this information and how will this type of cancer be treated?

-this type of cancer has a better prognosis and usually responds to hormonal therapy

A 72 y/o patient admitted to the med-surg unit tells the nurse that he has benign prostatic hyperplasia. Which question will the nurse ask?

-"are you having urinary incontinence or frequency at night?"

The nurse is teaching a group of women about prevention of toxic shock syndrome. What preventative measure does the nurse include?

-"change tampon every 3-6 hrs"

A patient calls to make an appointment for a routine pelvic exam which includes a pap smear. Which type of instructions does the nurse give the patient about preparing for the exam?

-"do not have intercourse for at least 24 hours before the exam"

The nurse is teaching a patient about the contraindications for hysteroscopy. What does the nurse telll the patient?

-"during the procedure, normal or abnormal cells can be pushed through the fallopian tubes and into the pelvic cavity; therefore pregnancy is a contraindication"

During clinical breast exam, the examiner observes a small mass in the breast. What is the most important item to include in the documentation of this finding?

-"face of the clock" location of the mass

The nurse is interviewing a patient who reports a discharge from his penis that started 3 days ago. What does the nurse ask the patient regarding this problem?

-"has your sexual partner(s) noticed a discharge?" -"does the discharge have an odor?" -"are you having any pain or burning during urination?" -"what is the consistency of the discharge?" -"can you describe the color of the discharge?"

A patient reports finding a mass in her breast 6 months ago. What question does the nurse ask related to possible metastases of a potential cancer?

-"have you noticed any joint or bone pain or any other changes in your body?"

The patient reports itching, change in vaginal discharge, and an odor. The nurse suspects that the patient has vulvovaginitis. Based on knowledge about common causes of vulvovaginitis, which questions would the nurse ask?

-"have you recently been taking antibiotics?" -"do you consistently wipe from front to back?" -"do you use tampons?" -"do you douche or use vaginal sprays?" -"have you had problems with vaginal yeast infections?"

A woman recently diagnosed with breast cancer confides, "i am going to use nutritional and herbal therapy instead of taking drugs and radiation that would make my hair fall out." What is the nurse's best response?

-"have you reviewed all treatment options with your HCP?"

The nurse must assign unlicensed assistive personnel to assist a patient who is undergoing brachytherapy for breast cancer treatment. What is the most important question that the nurse will ask the UAP prior to making the assignment?

-"is there any chance that you could be pregnant?"

The nurse is teaching the patient about breast self examination. Which correct actions are included in the procedure?

-"lie down on your back and place your right arm behind your head" -"use three different levels of pressure to feel all the breast tissue" -"move around the breast in an up and down pattern" -"repeat the exam on your left breast" -"stand in front of a mirror, press your hands firmly down on your hips, and observe breasts"

If the prostate gland is not functioning correctly, which statement typifies what the patient is most likely to report?

-"my wife and i have been trying to conceive a child for several years"

The nurse is giving instructions to a patient who is undergoing brachytherapy for cervical cancer. What information does the nurse include?

-"report any blood in the urine or severe diarrhea immediately"

A patient has just been informed by the HCP that she has specific BRCA 1 and BRCA 2 gene mutations. Which brochure would the nurse prepare for the patient?

-"risk factors and treatment for breast cancer"

A woman has had several discussions with the HCP about her risk for breast cancer. What would the nurse say in order to reinforce the role of prophylactic mastectomy on the risk for developing breast cancer?

-"the surgery reduces the risk for breast cancer"

What are the rationales for chemotherapy as a cancer treatment?

-Increases survival time for the patient -Systemic treatment for cancer cells that may have escaped from the primary tumor

The nurse is caring for a patient who had a laparoscopy. What is included in the postoperative care for this patient?

-administer oral analgesic for incisional pain -instruct the patient to change the small adhesive bandage as needed -teach the patient to observe the incision for signs of infection or hematoma

The nurse is designing a teaching plan for a patient who had surgery for breast cancer. What information does the nurse include in the plan?

-do not use lotions or ointments on the area -delay using deodorant under the affected arm until healing is complete -report any increased heat and tenderness of the area to the surgeon -begin active ROM exercises 1 week after surgery

What posprocedural instructions would the nurse give to a patient who just had a colposcopy?

-do not use tampons and abstain from sexual intercourse for at least a week

A patient receiving chemotherapy treatments reports fatigue, loss of energy, and experiencing an "emotional crisis every day and my hair is falling out". What does the nurse do first to help the pt adapt to body changes?

-encourage pt to ventilate feelings

The home health nurse is reviewing the patient's medication list and sees that the patient was given doxorubicin at the hospital. What gynecologic diagnosis would the nurse expect to see as part of the pt's history?

-endometrial cancer

Which disorder is strongly associated with prolonged exposure to estrogen without the protective effects of progesterone?

-endometrial cancer

What information would the nurse give to a sexually active 35 y/o woman about conventional papanicolaou (Pap) smear and human papillomavirus (HPV) testing?

-every 5 yrs is sufficient

The nurse is counseling a woman who was recently diagnosed with breast cancer. Which factor has the most influence on the choice for treatment?

-extent and location of metastases

A 23 y/o woman is diagnosed with chlamydial infection but is reluctant to spend the money for treatment because she is asymptomatic and does not have a job or health insurance. The nurse advises her that chlamydial infections can result in which condition?

-female infertility

A 22 y/o woman is being seen for a self detected mass in her right breast. Clinical examination reveals an oval-shaped, freely mobile, and rubbery lesion. What type of tumor is this most likely to be?

-fibroadenoma

The nurse is caring for a patient who had hysteroscopic surgery. The nurse is vigilant to assess for s/sx of which potential complications?

-fluid overload -embolism -perforation of uterus -hemorrhage -ureter injury

Regarding the diagnosis and treatment of breast cancer, what are important considerations for young women?

-genetic predisposition is a stronger risk factor for young women compared to older women -young women frequently have more aggressive forms of the disease -screening tools can be less effective because young women have denser breast tissue -early menopause, infertility, and sexual dysfunction are concerns for young women

In males who have breast cancer, what is the most typical presenting s/sx?

-hard, painless, subareolar mass

Which woman has the highest risk for developing cervical canceR?

-has an abnormal pap test result and a positive human papillomavirus test

The nurse is talking to a patient who is about to undergo a hysterosalpingogram. The patient discloses information that may not have been available to the HCP when the test was initially scheduled. Which disclosure could cause the provider to cancel or reschedule the test?

-has fever with malodorous vaginal discharge

The nurse is caring for a patient who had hysteroscopic surgery. the patient reports severe lower abdominal pain, appears pale, and has trouble focusing on the nurse's questions about the pain. Vital signs show T 98.6, HR 120, RR 24/min, BP 103/60. Which complication does the nurse suspect?

-hemorrhage

Which therapies would the nurse expect to use for a patient who is being treated for a rectocele?

-high fiber diet, stool softeners, laxatives

The nurse is assisting a patient who needs a pelvic examination. Which action will the nurse perform?

-instruct to empty the urinary bladder

The nurse hears in shift report that a 32 y/o patient had a prophylactic oophorectomy. What subjective symptoms does the nurse anticipate the patient would report?

-menopausal symptoms

A patient had breast reconstruction surgery two days ago. A Jackson-Pratt drain was placed to collect serosanguineous fluid. The nurse notes at 7am that the drainage container contains 150 mL. It was last emptied at 6am. What is the priority nursing intervention?

-notify the HCP about the amount and appearance of drainage for the past hour

The nurse is assessing a woman with very large breasts. In addition to the routine assessment of the breasts, what specific assessment will the nurse perform on this patient?

-observe underneath the breasts for fungal infection

The nurse is teaching a 24y/o patient about breast self examination. What does the nurse tell the patient about the best time to perform BSE?

-one week after her menstrual period

Females with an insufficient estrogen level should be assessed for which condition?

-osteoporosis

A 37 y/o patient reports abnormal vaginal bleeding not related to her menstrual cycle. The nurse would ask the patient about which associated symptoms?

-pain -change in bowel habits -abdominal fullness -urinary difficulties

Which classic symptom is indicative of invasive gynecologic cancer?

-painless vaginal bleeding unrelated to menses

Which test detects cancerous and precancerous cells of the cervix?

-pap smear

Which patient is describing an ominous sign associated with inflammatory breast cancer, which is a highly aggressive invasive breast cancer?

-patient reports breast pain and a rapidly growing breast lump

Which man has the greatest risk for developing prostate cancer?

-patient's father was diagnosed and treated for prostate cancer @ age 50

A 29 y/o patient has strictures and adhesions in her fallopian tubes. This can be the result of which condition?

-pelvic inflammatory disease

The nurse is caring for a 21-year-old client who had a left orchiectomy for testicular cancer on the previous day. Which nursing activity will be best to assign to an LPN/LVN? A. Educating the client about post-orchiectomy chemotherapy and radiation B. Administering the prescribed "as needed" (PRN) oxycodone to the client C. Teaching the client how to do testicular self-examination on the remaining testicle D. Assessing the client's knowledge level about post-orchiectomy fertility

B. Administering the prescribed "as needed" (PRN) oxycodone to the client

The nurse is interviewing a patient with erectile dysfunction. Which question would the nurse ask to assist the health care provider in differentiating the etiology as organic versus functional? A. "Have you ever had an elevated prostate specific antigen level?" B. "Do you ever have nocturnal emissions or morning erections?" C. "Have you tried any medications or therapies for erectile dysfunction?" D. "Do you have trouble passing urine or starting the stream?"

B. "Do you ever have nocturnal emissions or morning erections?"

The nurse is caring for a patient who had an open radical prostatectomy. During the assessment, the nurse notes that the penis and scrotum are swollen. What does the nurse do next? A. Notify the health care provider and monitor for any inability to void or increasing pain B. Elevate the scrotum and penis; apply ice intermittently to the area for 24-48 hours C. Assist the patient to increase mobility, especially early ambulation D. Observe the urethral meatus for redness and discharge and monitor urine output

B. Elevate the scrotum and penis; apply ice intermittently to the area for 24-48 hours

The patient had several diagnostic tests to evaluate lower urinary tract symptoms. Which finding suggests that the patient may have kidney disease? A. Elevated white blood cell count B. Elevated serum creatinine C. Elevated red blood cell count D. Elevated prostate-specific antigen

B. Elevated serum creatinine

The nurse notes that the patient has just started taking an alpha-blocker medication to treat benign prostatic hyperplasia. What instruction, related to the medication side effects, will the nurse give to unlicensed assistive personnel who will assist the patient with activities of daily living? A. Frequently offer the patient the urinal B. Have him sit up slowly and pause before standing C. Remind the patient to drink plenty of extra fluids D. Frequently check the linens for soiling and moisture

B. Have him sit up slowly and pause before standing

A nurse is caring for a client with a continuous bladder irrigation. Which is the most important nursing action? A. Monitoring urinary specific gravity to determine hydration B. Subtracting irrigant from output to determine urine volume C. Recording urinary output every hour to determine kidney function D. Obtaining a 24-hour urine specimen to determine urine concentration

B. Subtracting irrigant from output to determine urine volume

The nurse is caring for a patient who is taking finasteride, a 5-alpha reductase inhibitor. What question would the nurse ask to determine if the medication is having the desired therapeutic effect? A. "Have you had any discharge from your penis?" B. "Has your libido returned to the way it was before?" C. "Are you having any problems with urination?" D. "Have you gotten any relief from the testicular pain?"

C. "Are you having any problems with urination?"

A client has had a needle biopsy of the prostate gland using the transrectal approach. Which statement is most important to include in the client teaching plan? A. "The health care provider will call you about the test results" B. "Serious infections may occur as a complication of this test" C. "You will need to call the health care provider if you develop fever or chills" D. "It is normal to have a small amount of rectal bleeding after the test"

C. "You will need to call the health care provider if you develop fever or chills"

A patient reports having uncomfortable and unsettling episodes of "hot flashes" after receiving hormonal therapy for a prostate tumor. To alleviate this symptom, the nurse would obtain an order for which medication? A. Bisphosphonate drug such as pamidronate B. Antiandrogen drug such as biculatamide C. Hormonal inhibitor drug such as megestrol acetate D. Antimuscarinic agents such as tolterodine

C. Hormonal inhibitor drug such as megestrol acetate

The nurse is caring for an older patient who had an indwelling urinary catheter inserted after a transurethral resection of the prostate. The patient is intermittently confused, and picks at the IV tubing and the catheter. What should the nurse try first? A. Obtain an order to restrain the patient's hands and forearms B. Sedate the patient until the IV tube and catheter can be removed C. Inform the family that a family member will have to sit by the patient D. Give the patient a familiar object to hold, such as a family picture

D. Give the patient a familiar object to hold, such as a family picture

The nurse is preparing to assess an obese patient who reports subjective symptoms and urinary patterns associated with benign prostatic hyperplasia. Which technique does the nurse use to preform the physical assessment? A. Instruct the patient to undress from the waist down, then inspect and palpate the bladder B. Have the patient drink several large glasses of water and percuss the bladder C. Apply gentle pressure to the bladder to elicit urgency, then instruct the patient to void D. Instruct the patient to void and then use the bedside ultrasound bladder scanner

D. Instruct the patient to void and then use the bedside ultrasound bladder scanner

What role does vascular endothelial growth factor (VEGF) have in the metastasis of cancer? a. VEGF triggers capillary growth to ensure blood supply to the tumor b. Use of VEGF helps to stop the growth and spread of the primary tumor c. VEGF is a carcinogen that activates when cancer cells reach the vascular system d. For cancers with a genetic link, VEGF must be present before metastasis occurs

a. VEGF triggers capillary growth to ensure blood supply to the tumor

What instructions will the nurse give to the UAP regarding the hygienic care of a patient with neutropenia?

Mouth care and washing of the axillary and perianal regions must be done during the shift

The nurse works at an institution where pharmacogenomics is incorporated into the care of cancer patients. How does this newer approach impact nursing care?

Patient's risk for the more dangerous side effect is decreased

A patient is taking oprelvekin (Neumega). Which assessment data finding indicates that the therapy is working?

Platelet count is increasing

The charge nurse sees an order for IV chemotherapy. According to the Oncology Nursing Society, who should the charge nurse assign to administer the medication?

Registered nurse who completed an approved chemotherapy course

The nurse is caring for a patient for a 56-year-old woman who had a modified mastectomy for breast cancer. The woman jokes. "That breast was too saggy anyway. Good riddance to it." Later, the nurse sees the woman crying. What should the nurse do first?

Sit with the woman and encourage her to express her feelings and concerns

The nurse is reviewing the medication list of an older patient who is getting chemotherapy and filgrastim (Neupogen). Which intervention is the nurse most likely to use to facilitate the purpose of filgrastim (Neupogen)?

Teach patient, family, and all visitors about scrupulous hang hygiene

The nurse is caring for a patient who will receive stereotactic body radiotherapy. Which intervention is the nurse most likely to use in the care of this patient?

Teach the patient about the need for exact positioning during the treatment

The nurse reads in the patient's chart that the health care provider is concerned about myelosuppression. Which laboratory results will the nurse closely monitor and report to the provider? a. WBC count b. Serum potassium level c. RBC count d. Platelet count e. Serum sodium level f. Serum calcium level

a. WBC count c. RBC count d. Platelet count

The nurse is responsible for teaching the immunosuppressed patient and the family about health-promoting activities. Which information is correct?

Wash hands thoroughly with an antimicrobial soap

A patient is prescribed trastuzumab for breast cancer. What is the priority nursing intervention for this patient? a. Assess for signs of cardiotoxicity b. Assess for signs of bleeding c. Premedicate with an antiemetic d. Rotate injection sites

a. Assess for signs of cardiotoxicity

What information can be obtained by surgical staging? Select all a. Assessment of tumor size b. Number of tumors c. Sites of tumors d. Types of tumors e. Pattern spread of tumors f. Pain related to tumors

a. Assessment of tumor size b. Number of tumors c. Sites of tumors e. Pattern spread of tumors

The nurse reads a laboratory report indicating that the tissue sample of a patient is essentially neoplastic. How does the nurse interpret this report? a. Cell growth is abnormal and not needed for tissue replacement. b. The tissue specimen shows malignant cell growth. c. The parent cell was abnormal, but new growth is benign. d. Early cell death is inevitable because the morphology is abnormal.

a. Cell growth is abnormal and not needed for tissue replacement.

For women with genetic risk factors for breast cancer, which intervention would address one of the modifiable risk factors? a. Discuss strategies to avoid weight gain and obesity b. Encourage frequent genetic testing for tumors c. Have testing for BRCA1 and BRCA2 gene mutations d. Consider hormone replacement therapy

a. Discuss strategies to avoid weight gain and obesity

For a patient undergoing external radiation therapy, what do the nurse's instructions include? Select all a. Do not remove the markings b. Use lotions liberally to keep skin soft and moist c. Avoid direct skin exposure to sunlight for up to a year d. Use mild soap and water on the affected skin e. Gently rub treated areas to stimulate circulation f. Avoid wearing belts or clothing that binds the irradiated area

a. Do not remove the markings c. Avoid direct skin exposure to sunlight for up to a year d. Use mild soap and water on the affected skin f. Avoid wearing belts or clothing that binds the irradiated area

The nurse is designing a teaching plan for a patient who had surgery for breast cancer. What information does the nurse include in the plan? Select all a. Do not use lotions or ointments on the area b. Delay using deodorant until healing is complete c. Swelling and redness of the scar itself are normal and permanent d. Report any increased heat or tenderness to the area e. Wear loose pajamas at home for 6-8 weeks f. Begin active ROM exercises 1 week after surgery

a. Do not use lotions or ointments on the area b. Delay using deodorant until healing is complete d. Report any increased heat or tenderness to the area f. Begin active ROM exercises 1 week after surgery

Which factors are used to determine a cancer patient's absorbed radiation dose? Select all. a. Intensity of radiation exposure b. Proximity of radiation source to body c. Type of radiation particle d. Age of the patient during radiation therapy e. Overall health at time of radiation therapy f. Duration of radiation exposure

a. Intensity of radiation exposure b. Proximity of radiation source to body f. Duration of radiation exposure

Because chemotherapy drug dosage is based on total body surface area, the nurse should perform what assessment? a. Measure the patient's height and weight b. Compare the patient's weight to a nomogram c. Calculate the BMI d. Measure abdominal girth

a. Measure the patient's height and weight

Which patient has benign tumor cells that are the result of a small problem with cellular regulation? a. Patient is diagnosed with uterine fibroids b. Patient is advised that she has melanoma c. Patient is advised that he has a G1 tumor d. Patient is diagnosed with hairy cell leukemia

a. Patient is diagnosed with uterine fibroids

The nurse is caring for several patients who are receiving chemo. Which patient is the most likely to need transfer to the ICU? a. Patient receiving interleukin therapy for renal cell carcinoma develops widespread edema. b. Patient receiving estrogen therapy develops calf pain with redness and swelling c. Patient receiving vascular endothelial growth factor/receptor inhibitor has high BP d. Patient receiving an antiandrogen receptor develops gynecomastia

a. Patient receiving interleukin therapy for renal cell carcinoma develops widespread edema.

Why would the nurse encourage a patient to get a vaccine such as Gardasil or Cervarix? a. Protects against human papillomaviruses which are associated with genital cancers. b. Protects against associated with Epstein-Barr virus, which may contribute to Burkitt's lymphoma. c. Protects against hepatitis B virus, which may contribute to primary liver cancer. d. Protects against human lymphotropic virus, which may contribute to T-cell leukemia.

a. Protects against human papillomaviruses which are associated with genital cancers.

The nurse hears in report that the patient has xerostomia. Which teaching point does the nurse plan to review with the patient? a. Regular dental visits are essential because of increased risk for dental caries b. Use mild soap and apply unscented moisturizers to reduce itching sensation c. Avoid rigorous sports because bones are more prone to pathologic fractures d. Avoid direct sun exposure for at least 1 year because skin will be sensitive

a. Regular dental visits are essential because of increased risk for dental caries

What is the most typical schedule for radiation therapy? a. Small doses of radiation given on a daily basis for a set time period b. Large one-time dose of radiation given after completing chemotherapy c. Small doses of radiation given several days apart to minimize side effects d. Large doses administered monthly for a set period of months

a. Small doses of radiation given on a daily basis for a set time period

Which tumor, node, metastasis (TNM) staging classification would indicate the best prognosis for the patient's survival? a. T(IS)N(0)M(0) b. T(x)N(X)M(x) c. T(2)N(1)M(0) d. T(2)N(3)M(1)

a. T(IS)N(0)M(0)

Which tumor, node, metastasis (TNM) staging classification would indicate the BEST prognosis for the patient's survival? a. T15 N0 Mo b. Tx Nx Mx c. T2 N1 Mo d. T2 N3 M1

a. T15 N0 Mo

The nurse is caring for an older patient who is getting chemotherapy and filgrastim. Which intervention is the nurse most likely to use to facilitate the purpose of the filgrastim? a. Teach patient, family, and all visitors about meticulous hand hygiene b. Administer the filgrastim prior to chemo to prevent nausea c. Teach and assess for bleeding signs such as bruising or bleeding gums d. Assess the patient for fatigue and plan for periods of uninterrupted rest

a. Teach patient, family, and all visitors about meticulous hand hygiene

A patient is receiving radiation treatment by teletherapy. When does exposure to the patient create a risk for harmful radiation? a. The patient is never radioactive b. During the mechanical delivery of gamma rays c. For the first 24 to 48 hours after treatment d. Until the radiation source has decayed by one half-life

a. The patient is never radioactive

Which statement correctly describes metastatic tumors? a. They are caused by cells breaking off from the primary tumor b. They become less malignant over time c. They are usually less harmful than a primary tumor d. They become the tissue of the organ where they spread

a. They are caused by cells breaking off from the primary tumor

Which outcome statement indicates that the goal of cytoreductive surgery for cancer has been met? a. Tumor size has been decreased and chemotherapy is pending b. The noninvasive skin cancer was completely removed during surgery c. Subjective back pain has decreased since the removal of the tumor d. Incisional site of breast reconstruction shows no signs of infection

a. Tumor size has been decreased and chemotherapy is pending

Which factors determine the type of therapy for cancer? Select all. a. Type and location of cancer b. Overall health of the patient c. Whether the cancer has metastasized d. Previous lymph node biopsy e. Patient's gender f. Family history and genetics

a. Type and location of cancer b. Overall health of the patient c. Whether the cancer has metastasized f. Family history and genetics

What role does vascular endothelial growth factor (VEGF) have in the metastasis of cancer? a. VEGF triggers capillary growth to ensure blood supply to the tumor b. use of VEGF helps to stop the growth and spread of the primary tumor c. VDGF is a carcinogen that activates when cancer cells reach the vascular system d. for cancers with a genetic link, VEGF must be present before metastasis occurs

a. VEGF triggers capillary growth to ensure blood supply to the tumor

The nurse hears in report that the patient is diagnosed with glioblastoma. Which question is the most important to ask the off-going nurse? a. What is the patient's current mental status? b. Does the patient have leg pain during ambulation? c. Is the patient able to eat a normal diet? d. Does the patient have trouble passing urine?

a. What is the patient's current mental status?

Why would the nurse encourage a patient to get a vaccine such as Gardasil or Cervarix? a. protects against human papillomaviruses, which are associated with genital cancers b. protects against epstein-barr virus, which may contribute to burkitt's lymphoma c. protects against hepatitis B virus, which may contribute to primary liver cancer d. protects against hyman lymphotrophic virus, which may contribute to t-cell leukemia

a. protects against human papillomaviruses, which are associated with genital cancers

Which statement correctly describes metastatic tumors? a. they are caused by cells breaking off from the primary tumor b. they become less malignant over time c. they are usually less harmful than a primary tumor d. they become the tissue of the organ where they spread

a. they are caused by cells breaking off from the primary tumor

The nurse hears in report that the patient is diagnosed with glioglastoma. Which questions is the MOST important to ask the off-going nurse? a. what is the patient's current mental status b. does the patient have leg pain during ambulation c. is the patient able to eat a normal diet d. does the patient have trouble passing urine

a. what is the patient's current mental status

The nurse is preparing a brochure to inform patients about secondary prevention of cancer. Which information would be included? a. yearly mammography for women starting at the age of 45 b. chemo prevention with vitamin therapy c. removing colon polyps for cancer prophylaxis d. using sunscreen and hat when outdoors

a. yearly mammography for women starting at the age of 45

What is the minimum size for a detectable tumor? a. 1 millimeter b. 1 centimeter c. depends on type of tumor d. depends on site of tumor

b. 1 centimeter

The nurse hears in report that the patient has cachexia. Which assessment will the nurse plan to perform? a. Ability to ambulate independently b. Appetite and nutritional intake c. Mental status and cognition d. Sensation and pulses in extremities

b. Appetite and nutritional intake

The American Cancer Society reports that the cancer incidence and survival rate are related to which factors? a. Gender of patient and gender of family caregiver b. Availability of and access to health care services c. Belief that cancer is a chronic disorder d. Age at initiation of lifestyle modification

b. Availability of and access to health care services

Each chemo agent has a specific nadir. What is important for the nurse to do when giving combination therapy? a. Give two agents with similar nadirs b. Avoid giving agents with similar nadirs at the same time c. Watch for first agent's nadir and then give second agent d. Give two agents from different drug classes

b. Avoid giving agents with similar nadirs at the same time

An older patient is receiving epoetin alfa. Based on the knowledge that this med increases erythrocytes and many other types of blood cells, which abnormal assessment finding suggests the patient is experiencing an adverse effect of the medication? a. Temperature is 100.5*F b. BP is 160/90mmHg c. Patient has hemorrhagic cystitis d. Mucous membranes are dry

b. BP is 160/90mmHg

The nurse is reviewing the laboratory results from a postmenopausal woman being evaluated for a breast mass. What type of metastasis does the increased serum calcium and alkaline phosphatase levels suggest? a. Brain b. Bone c. Lung d. Liver

b. Bone

Which task is the nurse most likely to perform when caring for a patient with cancer? a. Informs a 36 yo woman about the initial diagnosis of breast cancer b. Explains recommendations for yearly mammograms to a 50 yo woman c. Suggests treatments based on staging of breast tumor to a 65 yo woman d. Advises a 23 yo woman to have surgery for breast cancer

b. Explains recommendations for yearly mammograms to a 50 yo woman

The health care provider informs the nurse that it is likely that the patient's cancer has invaded the bone marrow. Based on this information, the nurse will be vigilant for which signs and symptoms? Select all a. Nausea and vomiting b. Fatigue and weakness c. Decreasing WBCs d. Confusion with memory loss e. Bruises or other bleeding signs f. Tachycardia and shortness of breath

b. Fatigue and weakness c. Decreasing WBCs e. Bruises or other bleeding signs f. Tachycardia and shortness of breath

Which areas of the body contain cells that grow throughout the life span? Select all a. Heart b. Hair c. Brain d. Bone marrow e. Skin f. Lining of intestines

b. Hair d. Bone marrow e. Skin f. Lining of intestines

Which cancer patient is the most likely candidate for palliative surgery? a. Needs extensive cosmetic repair after treatment of neck cancer b. Has continuous vomiting because tumor is obstructing intestines c. Has a suspicious skin lesion that requires further investigation d. Has been treated for cancer and is currently asymptomatic

b. Has continuous vomiting because tumor is obstructing intestines

A young woman is suspected of having invasive breast cancer. Based on the types and frequencies, what is the most likely diagnosis? a, Fibrocystic breast condition b. Infiltrating ductal carcinoma c. Lobular carcinoma in situ d. Ductal carcinoma in situ

b. Infiltrating ductal carcinoma

Why do cancer cells spread throughout the body? SATA a. they enrich nutrients at the original site b. they have loose adherence c. they readily slip through blood vessel walls and tissue d. they do not respond to contact inhibition e. they are fragile and easily break apart f. they readily respond to signals for apoptosis

b. they have loose adherence c. they readily slip through blood vessel walls and tissue d. they do not respond to contact inhibition

What instructions will the nurse give to UAPs regarding the hygienic care of a patient with neutropenia? a. Do not enter the room unless absolutely necessary and then minimize time spent in the room b. Mouth care and washing of the axillary and perianal regions must be done during the shift c. If the patient seems very tired, assist with toileting but defer all other aspects of hygienic care d. Assist the patient to perform hygienic care according to the standard routine for all patients

b. Mouth care and washing of the axillary and perianal regions must be done during the shift

If apoptosis is occurring within a patient's body, what is the expected outcome of this physiologic process? a. Rapid growth of malignant tumor metastasizing throughout the body b. Organs have an adequate number of cells at their functional peak c. Normal tissue continues to function in an abnormal place d. Cells will initially resemble parent cells but will rapidly mutate

b. Organs have an adequate number of cells at their functional peak

Which patient has a condition that is a significant contraindication for photodynamic therapy? a. Patient has a history of frequent sunburn and is at risk for skin cancer b. Patient has a known tumor involvement of a major blood vessel c. Patient needs treatments that would involve the upper airways d. Patient had surgery for breast cancer several years ago

b. Patient has a known tumor involvement of a major blood vessel

What is the most common side effect of radiation? a. Altered taste sensation b. Radiodermatitis c. Nausea d. Fatigue

b. Radiodermatitis

The nurse is aware that the most common way for cancer to spread is bloodborne metastasis. In caring for a patient with cancer, what type of precautions would the nurse use? a. Contact isolation precautions b. Standard precautions c. Neutropenic precautions d. Droplet precautions

b. Standard precautions

Which patient circumstance would prompt the nurse to create a three-generation pedigree to more fully explore the possibility of genetic risk? a. Smoked for 20 years but quit 5 years ago b. Strong family history of colorectal cancer c. Male relatives with prostate problems d. Personal history of excessive sun exposure

b. Strong family history of colorectal cancer

What role do normal hormones and proteins such as insulin and estrogen play in the development of cancer? a. They prolong or delay the growth of cancer cells b. They can promote frequent division of cells c. They act like carcinogens under certain conditions d. They turn off the suppressor genes

b. They can promote frequent division of cells

Why do cancer cells spread throughout the body? Select all. a. They enrich nutrients at the original site b. They have loose adherence c. They readily slip through blood vessel walls and tissue d. They do not respond to contact inhibition e. They are fragile and easily break apart f. They readily respond to signals for apoptosis

b. They have loose adherence c. They readily slip through blood vessel walls and tissue d. They do not respond to contact inhibition

After surgery, a female has been told her breast tumor contains estrogen receptors. What is the clinical significance of this information and how will this type of cancer be treated? a. This is a triple negative breast cancer and additional surgery is the best option b. This type of cancer has a better prognosis and usually responds to hormonal therapy c. This tumor is localized; therefore, radiation therapy should effectively eradicate the cancer d. There are metastases, so long-term survival rate is low; systemic therapy is the only option

b. This type of cancer has a better prognosis and usually responds to hormonal therapy

Which patient with cancer has the greatest risk for infection? a. Recently diagnosed with breast cancer b. With neutropenia from leukemia c. With lung cancer who has a persistent cough d. Diagnosed with prostate cancer 3 years ago

b. With neutropenia from leukemia

the american cancer society reports that the cancer incidence and survival rate are related to which factors? a. gender of patient and gender of family caregiver b. availability of and access to health care services c. belief that cancer is a chronic disorder d. age at initiation of lifestyle modification

b. availability of and access to health care services

Which task is the nurse most likely to perform when caring for a patient with cancer? a. informs a 36 year old woman about the initial diagnosis of breast cancer b. explains recommendations for yearly mammograms to a 50 year old woman c. suggests treatments based on staging of breast tumor to a 65 year old woman d. advises a 23 year old woman to have surgery for breast cancer

b. explains recommendations for yearly mammograms to a 50 year old woman

Which areas of the body contain cells that grow throughout the life span? SATA a. heart b. hair c. brain d. bone marrow e. skin f. lining of intestines

b. hair d. bone marrow e. skin f. lining of intestines

The nurse is assessing a patient who is undergoing outpatient therapy for breast cancer. Which patient report causes the greatest concern because of possible metastasis to a common site? a. i don't seem to have a very good appetite b. my ribs hurt but i haven't had any injuries c. my skin is dry and it feels itchy and irritated d. i feel like i need to urinate all of the time

b. my ribs hurt but i haven't had any injuries

Benign cells have which characteristics? SATA a. contain few pairs of chromosomes? b. resemble the parent tissue c. growth is orderly with normal growth patterns d. perform their differentiated function e. invade other tissues f. continue to make fibronectin

b. resemble the parent tissue c. growth is orderly with normal growth patterns d. perform their differentiated function f. continue to make fibronectin

Cancer surveillance for high-risk women is used to detect cancer in its early stages and is referred to as what kind of prevention? a. primary b. secondary c. tertiary d. prophylactic

b. secondary

Which patient circumstance would prompt the nurse to create a three-generation pedigree to more fully explore the possibility of genetic risk? a. smoke for 20 years but quit 5 years ago b. strong family history of colorectal cancer c. male relatives with prostate problems d. personal history of excessive sun exposure

b. strong family history of colorectal cancer

Which lunch tray represents a diet that would decrease the risk of cancer? a. plain chicken breast on white bread b. vegetable plate with a bran muffin c. grilled cheese sandwich with fruit salad d. bacon cheeseburger with french fries

b. vegetable plate with a bran muffin

Which patient circumstance represents the normal physiologic progress of mitosis and cellular regulation? a. A 25 yo woman is diagnosed with endometriosis b. A 45 yo woman notices several skin tags on her neck c. A 35 yo male has ulcer disease that is slowly resolving d. A 65 yo man has a benign tumor that seems to be enlarging

c. A 35 yo male has ulcer disease that is slowly resolving

A patient with colon cancer asks, "Why does everyone keep insisting that I eat so much? I'm not hungry and I've been overweight my whole life." What response is the most appropriate? a. What would you like to eat? I can get you something that you will enjoy b. The cancer may spread to your stomach so you should eat while you still can c. Cancer in the intestinal tract may increase metabolic rate and needs for nutrients d. Well, you don't have to eat if you don't want to, but eating will help your body to heal

c. Cancer in the intestinal tract may increase metabolic rate and needs for nutrients

Which information can be obtained from grading a tumor? a. Genetic linkage to the cancer b. Location and origin of metastasis c. Evaluating prognosis and appropriate therapy d. How long the cancer has been present

c. Evaluating prognosis and appropriate therapy

Why does the nurse wear a dosimeter when providing care to a patient receiving brachytherapy? a. Indicates special expertise in radiation therapy b. Protects the nurse from absorbing radiation c. Measures the nurse's exposure to radiation d. Ensures that the radiation dosage is accurate

c. Measures the nurse's exposure to radiation

The nurse hears in shift report that a 32 yo patient had prophylactic oophorectomy. What subjective symptoms does the nurse anticipate that the patient would report? a. Swelling of upper arms b. Tenderness of breasts c. Menopausal symptoms d. Nausea related to medications

c. Menopausal symptoms

African Americans have the highest rate of cancer and the highest death rate from cancer. Which intervention targets the most likely explanation for this disparity? a. Increase local efforts to dispense cancer information to this vulnerable group b. Develop educational materials that are culturally sensitive toward African Americans c. Provide referral information to health care facilities that are affordable and accessible. d. Continue research that further clarifies the genetic or racial risk for cancer.

c. Provide referral information to health care facilities that are affordable and accessible.

Which biologic process demonstrates the differentiated function of red blood cells (RBCs)? a. RBCs float freely through the circulatory system b. RBCs die according to programmed cell death c. RBCs make hemoglobin, which carries oxygen d. RBCs are formed with 23 pairs of chromosomes

c. RBCs make hemoglobin, which carries oxygen

Which biologic process demonstrates the differentiated function of red blood cells (RBCs)? a. RBCs float freely through the circulatory system. b. RBCs die according to programmed cell death. c. RBCs make hemoglobin, which carries oxygen. d. RBCs are formed with 23 pairs of chromosomes.

c. RBCs make hemoglobin, which carries oxygen.

A patient is on a newer protocol, dose-dense chemotherapy. Which factor is most likely to contribute to patient noncompliance if the nurse fails to educate the patient and the family? a. Treatment is expensive and less likely to be covered by insurance b. Length of therapy is prolonged and progress is slow to manifest c. Side effects are likely to be more intense and unpleasant d. Medication administration is painful and pain does not respond to medications

c. Side effects are likely to be more intense and unpleasant

The nurse in the radiation department is caring for a patient who will receive stereotactic body radiotherapy. Which intervention is the nurse most likely to use in the care of this patient? a. Remind the patient that no pregnant visitors should come for several days b. Dispose of radioactive urine and stool so that self and others are not exposed c. Teach the patient about the need for exact positioning during the treatment d. Assess the patient for history of allergies to iodine or contrast media

c. Teach the patient about the need for exact positioning during the treatment

The nurse is talking to a young woman who "is using a tanning salon, because it is a safer way to get a tan". What is the best response? a. Even if you use a tanning salon, you should still use a sunscreen. b. Tanning salons are safer because exposure to radiation is very controlled. c. Ultraviolet radiation from sun exposure or tanning salons can cause skin cancer. d. Ionizing radiation is dangerous, but tanning salons use ultraviolet radiation.

c. Ultraviolet radiation from sun exposure or tanning salons can cause skin cancer.

The nurse is supervising a nursing student who is giving care to a patient with a sealed implant. The nurse would intervene if the student performed which action? a. Places a "caution: radioactive material" sign on the door b. Wears a dosimeter film badge at all times while caring for the patient c. Wears a lead apron while providing care and turns away from the patient d. Saves all dressings and bed linens in the patient's room

c. Wears a lead apron while providing care and turns away from the patient

Which patient circumstance represents the normal physiologic progress of mitosis and cellular regulation? a. a 25 year old woman is diagnosed with endometriosis b. a 45 year old woman notices several skin tags on her neck c. a 35 year old male has ulcer disease that is slowly resolving d. a 65 year old man has a benign tumor that seems to be enlarging

c. a 35 year old male has ulcer disease that is slowly resolving

What is the major side effect that limits the dose of chemo? a. nausea and vomiting b. peripheral neuropathy c. bone marrow suppression d. "chemo brain"

c. bone marrow suppression

Which information can be obtained from grading a tumor? a. genetic linkage to the cancer b. location and origin of metastasis c. evaluating prognosis and appropriate therapy d. how long the cancer has been present

c. evaluating prognosis and appropriate therapy

Which cells would normally not produce fibronectin? a. normal nerve cells b. normal cardiac muscle cells c. normal red blood cells d. cells that are undergoing normal mitosis

c. normal red blood cells

Which cells would normally not produce fibronectin? a. normal nerve cells b. normal cardiac muscle cells c. normal red blood cells d. cells that are undergoing normal mitosis

c. normal red blood cells

African Americans have the highest rate of cancer and the highest death rate from cancer. Which intervention targets the MOST LIKELY explanation of this disparity? a. increase local efforts to dispense cancer information to this vulnerable group b. develop educational materials that are culturally sensitive toward African Americans c. provide referral information to health care facilities that are affordable and accessible d. continue research that further clarifies the genetic or racial risk

c. provide referral information to health care facilities that are affordable and accessible

The nurse is talking to a young woman who "is using a tanning salon, because it is a safer way to get a tan than lying the sun." what is the BEST response? a. even if you use a tanning salon, you should still use a sunscreen b. tanning salons are safer because exposure to radiation is very controlled c. ultraviolet radiation from sun exposure or tanning salons can cause skin cancer d. ionizing radiation is dangerous, but tanning salons use ultraviolet radiation

c. ultraviolet radiation from sun exposure or tanning salons can cause skin cancer

The nurse is talking to a young athlete who needs lung removal for treatment of lung cancer. Which statement best indicates that the patient is coping with the uncertainty of cancer and long-term impact on his physical activities? a. "If I delay the surgery, I could still compete for a couple of months" b. "My coach says I might be able to compete even with one lung" c. "Competing in sports is important to me, and eventually I will recover" d. "I love to compete in sports, but I like to do a lot of other things too"

d. "I love to compete in sports, but I like to do a lot of other things too"

Which person has the greatest risk for developing cancer? a. 10 yo African American with allergic asthma b. 32 yo Asian immigrant with low income c. 23 yo white American who has type 1 diabetes d. 62 yo African American who had an organ transplant

d. 62 yo African American who had an organ transplant

Which cancer patient has the highest risk to develop sepsis? a. 34 yo patient who has received high dose radiation to the upper chest area b. 66 yo patient with hypercalcemia and dehydration c. 53 yo patient with small cell lung cancer and hyponatremia d. 82 yo patient with neutropenia and a low grade fever

d. 82 yo patient with neutropenia and a low grade fever

A patient diagnosed with bone cancer reports fatigue, loss of appetite, and constipation. Which lab result does the nurse report immediately? a. Potassium 4.2 b. Magnesium 2.0 c. Sodium 140 d. Calcium 10.5

d. Calcium 10.5

Which biologic process demonstrates that there is a problem with cellular regulation? a. Living cells spend most of their time in G0 state b. Mitosis occurs to replace damaged tissue c. Cyclin activity is balanced by suppressor genes d. Cells continue to divide without contact inhibition

d. Cells continue to divide without contact inhibition

The nurse is counseling a woman who was recently diagnosed with breast cancer. Which factor has the most influence on the choice for her treatment? a. Age at the time of diagnosis b. Overall health status c. Personal choice and self-care capacity d. Extent and location of metastases

d. Extent and location of metastases

The patient has thyroid cancer and will be treated with injection of the radionuclide iodine-131 (brachytherapy). Which guideline is the most relevant when instructing unlicensed assistive personnel on how to assist the patient with hygiene and activities of daily living? a. Oncology Nursing Society practice guidelines b. American Cancer Society treatment guidelines c. Institutional evidence-based policies for infection control d. Institutional policies for handling body fluids and wastes

d. Institutional policies for handling body fluids and wastes

When is the best time to perform BSE? a. Day before menstrual flow is due b. Third day after menstrual flow starts c. Whenever ovulation occurs d. One week after menstrual period

d. One week after menstrual period

Based on the "inverse square law" for radiation exposure, which patient received the smallest radiation dose? a. Received radiation dose at a distance of 0.5 meter b. Received radiation dose at a distance of 1 meter c. Received radiation dose at a distance of 2.5 meters d. Received radiation dose at a distance of 3 meters

d. Received radiation dose at a distance of 3 meters

The nurse is caring for a 56 yo woman who had a modified mastectomy for breast cancer. The woman jokes, "That breast was too saggy anyway. Good riddance to it." Later, the nurse sees the woman crying. What should the nurse do first? a. Encourage the woman to accept body changes by looking at the surgical site. b. Suggest participation in a support group sponsored by the american cancer society c. Invite a breast cancer survivor who successfully coped with mastectomy d. Sit with the woman and encourage her to express her feelings and concerns

d. Sit with the woman and encourage her to express her feelings and concerns

Which patient report should be investigated as one of the seven warning signs of cancer? a. Soreness and stiffness to joints in the morning b. Abdominal pain related to irregular meals c. Redness to skin with pain after sun exposure d. Sore on nipple present for several months

d. Sore on nipple present for several months

Which features are specific to cancer cells? a. They grow very slowly but eventually harm the body b. They have a small, fragile nucleus that is easily damaged c. They produce fibronectin, which strengthens the cell wall d. They have an unlimited life span and spread easily

d. They have an unlimited life span and spread easily

From a primary prevention perspective, what is the most important information that the nurse should emphasize when teaching patients about tobacco and cancer risk? a. Risk for cancer increases when tobacco and alcohol are both used. b. Tobacco use is linked to many different types of cancer. c. Risk for cancer depends on the amount of tobacco used. d. Tobacco is the single most preventable source of carcinogenesis.

d. Tobacco is the single most preventable source of carcinogenesis.

The patient is prescribed a biologic response modifier, leukine. Which outcome statement about the medication therapy reflects the concept of immunity? a. Electrolyte levels are improving and there is no edema b. Erythrocytes are increased and fatigue is resolving c. Platelet count is normalizing and there are no signs of bleeding d. White cell count is improving and there are no signs of infection

d. White cell count is improving and there are no signs of infection

From a primary prevention perspective, what is the MOST important information that the nurse should emphasize when teaching patients about tobacco and cancer risk? a. risk for cancer increases when tobacco and alcohol are both used b. tobacco use in linked to many different types of cancer c. risk for cancer depends on the amount of tobacco used d. tobacco is the single most preventable source of carcinogenesis

d. tobacco is the single most preventable source of carcinogenesis


Kaugnay na mga set ng pag-aaral

Chapter 14: Spinal Cord Gross Anatomy

View Set

MEDSURG II: Saunders Medsurg Oncology

View Set